Taking a derivative of a magnitude of a vector












1














I have found several questions including this as a step in the explanation, but have not been able to find an explicit explanation of how to take a derivative of a magnitude of a vector.



I am trying to take the derivative d/dt ||r'(t)|| but don't know how to address the magnitude signs.










share|cite|improve this question







New contributor




icesk8er is a new contributor to this site. Take care in asking for clarification, commenting, and answering.
Check out our Code of Conduct.




















  • Do you have a formula for $r(t)$ and for $r'(t)$? If so get the formula for $||r'(t)||$ which will be a 1-var function of $t,$ then do derivative of that.
    – coffeemath
    Jan 4 at 21:34






  • 1




    What is your norm? The one derived from the dot product?
    – mathcounterexamples.net
    Jan 4 at 21:36










  • @mathcounterexamples.net r(t) is a path -- what would the norm derived from the dot product be?
    – icesk8er
    Jan 4 at 21:44
















1














I have found several questions including this as a step in the explanation, but have not been able to find an explicit explanation of how to take a derivative of a magnitude of a vector.



I am trying to take the derivative d/dt ||r'(t)|| but don't know how to address the magnitude signs.










share|cite|improve this question







New contributor




icesk8er is a new contributor to this site. Take care in asking for clarification, commenting, and answering.
Check out our Code of Conduct.




















  • Do you have a formula for $r(t)$ and for $r'(t)$? If so get the formula for $||r'(t)||$ which will be a 1-var function of $t,$ then do derivative of that.
    – coffeemath
    Jan 4 at 21:34






  • 1




    What is your norm? The one derived from the dot product?
    – mathcounterexamples.net
    Jan 4 at 21:36










  • @mathcounterexamples.net r(t) is a path -- what would the norm derived from the dot product be?
    – icesk8er
    Jan 4 at 21:44














1












1








1


0





I have found several questions including this as a step in the explanation, but have not been able to find an explicit explanation of how to take a derivative of a magnitude of a vector.



I am trying to take the derivative d/dt ||r'(t)|| but don't know how to address the magnitude signs.










share|cite|improve this question







New contributor




icesk8er is a new contributor to this site. Take care in asking for clarification, commenting, and answering.
Check out our Code of Conduct.











I have found several questions including this as a step in the explanation, but have not been able to find an explicit explanation of how to take a derivative of a magnitude of a vector.



I am trying to take the derivative d/dt ||r'(t)|| but don't know how to address the magnitude signs.







multivariable-calculus vectors






share|cite|improve this question







New contributor




icesk8er is a new contributor to this site. Take care in asking for clarification, commenting, and answering.
Check out our Code of Conduct.











share|cite|improve this question







New contributor




icesk8er is a new contributor to this site. Take care in asking for clarification, commenting, and answering.
Check out our Code of Conduct.









share|cite|improve this question




share|cite|improve this question






New contributor




icesk8er is a new contributor to this site. Take care in asking for clarification, commenting, and answering.
Check out our Code of Conduct.









asked Jan 4 at 21:28









icesk8ericesk8er

83




83




New contributor




icesk8er is a new contributor to this site. Take care in asking for clarification, commenting, and answering.
Check out our Code of Conduct.





New contributor





icesk8er is a new contributor to this site. Take care in asking for clarification, commenting, and answering.
Check out our Code of Conduct.






icesk8er is a new contributor to this site. Take care in asking for clarification, commenting, and answering.
Check out our Code of Conduct.












  • Do you have a formula for $r(t)$ and for $r'(t)$? If so get the formula for $||r'(t)||$ which will be a 1-var function of $t,$ then do derivative of that.
    – coffeemath
    Jan 4 at 21:34






  • 1




    What is your norm? The one derived from the dot product?
    – mathcounterexamples.net
    Jan 4 at 21:36










  • @mathcounterexamples.net r(t) is a path -- what would the norm derived from the dot product be?
    – icesk8er
    Jan 4 at 21:44


















  • Do you have a formula for $r(t)$ and for $r'(t)$? If so get the formula for $||r'(t)||$ which will be a 1-var function of $t,$ then do derivative of that.
    – coffeemath
    Jan 4 at 21:34






  • 1




    What is your norm? The one derived from the dot product?
    – mathcounterexamples.net
    Jan 4 at 21:36










  • @mathcounterexamples.net r(t) is a path -- what would the norm derived from the dot product be?
    – icesk8er
    Jan 4 at 21:44
















Do you have a formula for $r(t)$ and for $r'(t)$? If so get the formula for $||r'(t)||$ which will be a 1-var function of $t,$ then do derivative of that.
– coffeemath
Jan 4 at 21:34




Do you have a formula for $r(t)$ and for $r'(t)$? If so get the formula for $||r'(t)||$ which will be a 1-var function of $t,$ then do derivative of that.
– coffeemath
Jan 4 at 21:34




1




1




What is your norm? The one derived from the dot product?
– mathcounterexamples.net
Jan 4 at 21:36




What is your norm? The one derived from the dot product?
– mathcounterexamples.net
Jan 4 at 21:36












@mathcounterexamples.net r(t) is a path -- what would the norm derived from the dot product be?
– icesk8er
Jan 4 at 21:44




@mathcounterexamples.net r(t) is a path -- what would the norm derived from the dot product be?
– icesk8er
Jan 4 at 21:44










3 Answers
3






active

oldest

votes


















2














I will assume that you are working with the euclidian norm and the dot product. Let $v(t)$ be a vector:
$$v(t) cdot v(t)=|v(t)|^2$$
And
$$frac{mathrm{d}|v(t)|^2}{mathrm{d}t}=2|v(t)|frac{mathrm{d}|v(t)|}{mathrm{d}t}$$
Which implies that
$$begin{align}
frac{mathrm{d}|v(t)|}{mathrm{d}t}&=frac{1}{2|v(t)|}frac{mathrm{d}|v(t)|^2}{mathrm{d}t}\
&=frac{1}{2|v(t)|}frac{mathrm{d}(v(t) cdot v(t))}{mathrm{d}t}\
&=frac{1}{2|v(t)|}left(v(t) cdot v'(t)+v'(t) cdot v(t)right)\
&=frac{1}{2|v(t)|}left(2v(t) cdot v'(t)right)\
&=frac{v(t) cdot v'(t)}{|v(t)|}
end{align}$$






share|cite|improve this answer





















  • (If anyone is unfamiliar with the notation, $v^prime(t) = dot{v}(t) = frac{d v(t)}{d t}$.)
    – Nominal Animal
    Jan 5 at 16:12



















1














$$
eqalign{
& left| {r(t)} right|^2 = r_{,x} (t)^2 + r_{,y} (t)^2 + cdots cr
& 2left| {r(t)} right|{d over {dt}}left| {r(t)} right| = 2left( {r_{,x} (t)r_{,x} '(t) + r_{,y} (t)r_{,y} '(t) + cdots } right) cr
& {d over {dt}}left| {r(t)} right| = {{r_{,x} (t)r_{,x} '(t) + r_{,y} (t)r_{,y} '(t) + cdots } over {left| {r(t)} right|}} cr}
$$






share|cite|improve this answer





























    0














    If the norm is derived from the dot product $langle cdot , cdot rangle$, then



    $$frac{dVert r^prime(t) Vert}{dt} = frac{langle r^prime(t),r^{prime prime}(t)rangle}{Vert r^prime(t)Vert}.$$






    share|cite|improve this answer





















    • Could you explain how you got that?
      – icesk8er
      Jan 4 at 21:52










    • Using the chain rule. A more simple way is to use the coordinates and the fact that for a vector $x=(x_1, dots, x_n)$ (in dimension $n$), you have $Vert x Vert = sqrt{x_1^2 + dots +x_n^2}$
      – mathcounterexamples.net
      Jan 4 at 21:56













    Your Answer





    StackExchange.ifUsing("editor", function () {
    return StackExchange.using("mathjaxEditing", function () {
    StackExchange.MarkdownEditor.creationCallbacks.add(function (editor, postfix) {
    StackExchange.mathjaxEditing.prepareWmdForMathJax(editor, postfix, [["$", "$"], ["\\(","\\)"]]);
    });
    });
    }, "mathjax-editing");

    StackExchange.ready(function() {
    var channelOptions = {
    tags: "".split(" "),
    id: "69"
    };
    initTagRenderer("".split(" "), "".split(" "), channelOptions);

    StackExchange.using("externalEditor", function() {
    // Have to fire editor after snippets, if snippets enabled
    if (StackExchange.settings.snippets.snippetsEnabled) {
    StackExchange.using("snippets", function() {
    createEditor();
    });
    }
    else {
    createEditor();
    }
    });

    function createEditor() {
    StackExchange.prepareEditor({
    heartbeatType: 'answer',
    autoActivateHeartbeat: false,
    convertImagesToLinks: true,
    noModals: true,
    showLowRepImageUploadWarning: true,
    reputationToPostImages: 10,
    bindNavPrevention: true,
    postfix: "",
    imageUploader: {
    brandingHtml: "Powered by u003ca class="icon-imgur-white" href="https://imgur.com/"u003eu003c/au003e",
    contentPolicyHtml: "User contributions licensed under u003ca href="https://creativecommons.org/licenses/by-sa/3.0/"u003ecc by-sa 3.0 with attribution requiredu003c/au003e u003ca href="https://stackoverflow.com/legal/content-policy"u003e(content policy)u003c/au003e",
    allowUrls: true
    },
    noCode: true, onDemand: true,
    discardSelector: ".discard-answer"
    ,immediatelyShowMarkdownHelp:true
    });


    }
    });






    icesk8er is a new contributor. Be nice, and check out our Code of Conduct.










    draft saved

    draft discarded


















    StackExchange.ready(
    function () {
    StackExchange.openid.initPostLogin('.new-post-login', 'https%3a%2f%2fmath.stackexchange.com%2fquestions%2f3062135%2ftaking-a-derivative-of-a-magnitude-of-a-vector%23new-answer', 'question_page');
    }
    );

    Post as a guest















    Required, but never shown

























    3 Answers
    3






    active

    oldest

    votes








    3 Answers
    3






    active

    oldest

    votes









    active

    oldest

    votes






    active

    oldest

    votes









    2














    I will assume that you are working with the euclidian norm and the dot product. Let $v(t)$ be a vector:
    $$v(t) cdot v(t)=|v(t)|^2$$
    And
    $$frac{mathrm{d}|v(t)|^2}{mathrm{d}t}=2|v(t)|frac{mathrm{d}|v(t)|}{mathrm{d}t}$$
    Which implies that
    $$begin{align}
    frac{mathrm{d}|v(t)|}{mathrm{d}t}&=frac{1}{2|v(t)|}frac{mathrm{d}|v(t)|^2}{mathrm{d}t}\
    &=frac{1}{2|v(t)|}frac{mathrm{d}(v(t) cdot v(t))}{mathrm{d}t}\
    &=frac{1}{2|v(t)|}left(v(t) cdot v'(t)+v'(t) cdot v(t)right)\
    &=frac{1}{2|v(t)|}left(2v(t) cdot v'(t)right)\
    &=frac{v(t) cdot v'(t)}{|v(t)|}
    end{align}$$






    share|cite|improve this answer





















    • (If anyone is unfamiliar with the notation, $v^prime(t) = dot{v}(t) = frac{d v(t)}{d t}$.)
      – Nominal Animal
      Jan 5 at 16:12
















    2














    I will assume that you are working with the euclidian norm and the dot product. Let $v(t)$ be a vector:
    $$v(t) cdot v(t)=|v(t)|^2$$
    And
    $$frac{mathrm{d}|v(t)|^2}{mathrm{d}t}=2|v(t)|frac{mathrm{d}|v(t)|}{mathrm{d}t}$$
    Which implies that
    $$begin{align}
    frac{mathrm{d}|v(t)|}{mathrm{d}t}&=frac{1}{2|v(t)|}frac{mathrm{d}|v(t)|^2}{mathrm{d}t}\
    &=frac{1}{2|v(t)|}frac{mathrm{d}(v(t) cdot v(t))}{mathrm{d}t}\
    &=frac{1}{2|v(t)|}left(v(t) cdot v'(t)+v'(t) cdot v(t)right)\
    &=frac{1}{2|v(t)|}left(2v(t) cdot v'(t)right)\
    &=frac{v(t) cdot v'(t)}{|v(t)|}
    end{align}$$






    share|cite|improve this answer





















    • (If anyone is unfamiliar with the notation, $v^prime(t) = dot{v}(t) = frac{d v(t)}{d t}$.)
      – Nominal Animal
      Jan 5 at 16:12














    2












    2








    2






    I will assume that you are working with the euclidian norm and the dot product. Let $v(t)$ be a vector:
    $$v(t) cdot v(t)=|v(t)|^2$$
    And
    $$frac{mathrm{d}|v(t)|^2}{mathrm{d}t}=2|v(t)|frac{mathrm{d}|v(t)|}{mathrm{d}t}$$
    Which implies that
    $$begin{align}
    frac{mathrm{d}|v(t)|}{mathrm{d}t}&=frac{1}{2|v(t)|}frac{mathrm{d}|v(t)|^2}{mathrm{d}t}\
    &=frac{1}{2|v(t)|}frac{mathrm{d}(v(t) cdot v(t))}{mathrm{d}t}\
    &=frac{1}{2|v(t)|}left(v(t) cdot v'(t)+v'(t) cdot v(t)right)\
    &=frac{1}{2|v(t)|}left(2v(t) cdot v'(t)right)\
    &=frac{v(t) cdot v'(t)}{|v(t)|}
    end{align}$$






    share|cite|improve this answer












    I will assume that you are working with the euclidian norm and the dot product. Let $v(t)$ be a vector:
    $$v(t) cdot v(t)=|v(t)|^2$$
    And
    $$frac{mathrm{d}|v(t)|^2}{mathrm{d}t}=2|v(t)|frac{mathrm{d}|v(t)|}{mathrm{d}t}$$
    Which implies that
    $$begin{align}
    frac{mathrm{d}|v(t)|}{mathrm{d}t}&=frac{1}{2|v(t)|}frac{mathrm{d}|v(t)|^2}{mathrm{d}t}\
    &=frac{1}{2|v(t)|}frac{mathrm{d}(v(t) cdot v(t))}{mathrm{d}t}\
    &=frac{1}{2|v(t)|}left(v(t) cdot v'(t)+v'(t) cdot v(t)right)\
    &=frac{1}{2|v(t)|}left(2v(t) cdot v'(t)right)\
    &=frac{v(t) cdot v'(t)}{|v(t)|}
    end{align}$$







    share|cite|improve this answer












    share|cite|improve this answer



    share|cite|improve this answer










    answered Jan 4 at 21:57









    BotondBotond

    5,5882732




    5,5882732












    • (If anyone is unfamiliar with the notation, $v^prime(t) = dot{v}(t) = frac{d v(t)}{d t}$.)
      – Nominal Animal
      Jan 5 at 16:12


















    • (If anyone is unfamiliar with the notation, $v^prime(t) = dot{v}(t) = frac{d v(t)}{d t}$.)
      – Nominal Animal
      Jan 5 at 16:12
















    (If anyone is unfamiliar with the notation, $v^prime(t) = dot{v}(t) = frac{d v(t)}{d t}$.)
    – Nominal Animal
    Jan 5 at 16:12




    (If anyone is unfamiliar with the notation, $v^prime(t) = dot{v}(t) = frac{d v(t)}{d t}$.)
    – Nominal Animal
    Jan 5 at 16:12











    1














    $$
    eqalign{
    & left| {r(t)} right|^2 = r_{,x} (t)^2 + r_{,y} (t)^2 + cdots cr
    & 2left| {r(t)} right|{d over {dt}}left| {r(t)} right| = 2left( {r_{,x} (t)r_{,x} '(t) + r_{,y} (t)r_{,y} '(t) + cdots } right) cr
    & {d over {dt}}left| {r(t)} right| = {{r_{,x} (t)r_{,x} '(t) + r_{,y} (t)r_{,y} '(t) + cdots } over {left| {r(t)} right|}} cr}
    $$






    share|cite|improve this answer


























      1














      $$
      eqalign{
      & left| {r(t)} right|^2 = r_{,x} (t)^2 + r_{,y} (t)^2 + cdots cr
      & 2left| {r(t)} right|{d over {dt}}left| {r(t)} right| = 2left( {r_{,x} (t)r_{,x} '(t) + r_{,y} (t)r_{,y} '(t) + cdots } right) cr
      & {d over {dt}}left| {r(t)} right| = {{r_{,x} (t)r_{,x} '(t) + r_{,y} (t)r_{,y} '(t) + cdots } over {left| {r(t)} right|}} cr}
      $$






      share|cite|improve this answer
























        1












        1








        1






        $$
        eqalign{
        & left| {r(t)} right|^2 = r_{,x} (t)^2 + r_{,y} (t)^2 + cdots cr
        & 2left| {r(t)} right|{d over {dt}}left| {r(t)} right| = 2left( {r_{,x} (t)r_{,x} '(t) + r_{,y} (t)r_{,y} '(t) + cdots } right) cr
        & {d over {dt}}left| {r(t)} right| = {{r_{,x} (t)r_{,x} '(t) + r_{,y} (t)r_{,y} '(t) + cdots } over {left| {r(t)} right|}} cr}
        $$






        share|cite|improve this answer












        $$
        eqalign{
        & left| {r(t)} right|^2 = r_{,x} (t)^2 + r_{,y} (t)^2 + cdots cr
        & 2left| {r(t)} right|{d over {dt}}left| {r(t)} right| = 2left( {r_{,x} (t)r_{,x} '(t) + r_{,y} (t)r_{,y} '(t) + cdots } right) cr
        & {d over {dt}}left| {r(t)} right| = {{r_{,x} (t)r_{,x} '(t) + r_{,y} (t)r_{,y} '(t) + cdots } over {left| {r(t)} right|}} cr}
        $$







        share|cite|improve this answer












        share|cite|improve this answer



        share|cite|improve this answer










        answered Jan 4 at 22:01









        G CabG Cab

        18.1k31237




        18.1k31237























            0














            If the norm is derived from the dot product $langle cdot , cdot rangle$, then



            $$frac{dVert r^prime(t) Vert}{dt} = frac{langle r^prime(t),r^{prime prime}(t)rangle}{Vert r^prime(t)Vert}.$$






            share|cite|improve this answer





















            • Could you explain how you got that?
              – icesk8er
              Jan 4 at 21:52










            • Using the chain rule. A more simple way is to use the coordinates and the fact that for a vector $x=(x_1, dots, x_n)$ (in dimension $n$), you have $Vert x Vert = sqrt{x_1^2 + dots +x_n^2}$
              – mathcounterexamples.net
              Jan 4 at 21:56


















            0














            If the norm is derived from the dot product $langle cdot , cdot rangle$, then



            $$frac{dVert r^prime(t) Vert}{dt} = frac{langle r^prime(t),r^{prime prime}(t)rangle}{Vert r^prime(t)Vert}.$$






            share|cite|improve this answer





















            • Could you explain how you got that?
              – icesk8er
              Jan 4 at 21:52










            • Using the chain rule. A more simple way is to use the coordinates and the fact that for a vector $x=(x_1, dots, x_n)$ (in dimension $n$), you have $Vert x Vert = sqrt{x_1^2 + dots +x_n^2}$
              – mathcounterexamples.net
              Jan 4 at 21:56
















            0












            0








            0






            If the norm is derived from the dot product $langle cdot , cdot rangle$, then



            $$frac{dVert r^prime(t) Vert}{dt} = frac{langle r^prime(t),r^{prime prime}(t)rangle}{Vert r^prime(t)Vert}.$$






            share|cite|improve this answer












            If the norm is derived from the dot product $langle cdot , cdot rangle$, then



            $$frac{dVert r^prime(t) Vert}{dt} = frac{langle r^prime(t),r^{prime prime}(t)rangle}{Vert r^prime(t)Vert}.$$







            share|cite|improve this answer












            share|cite|improve this answer



            share|cite|improve this answer










            answered Jan 4 at 21:50









            mathcounterexamples.netmathcounterexamples.net

            25.3k21953




            25.3k21953












            • Could you explain how you got that?
              – icesk8er
              Jan 4 at 21:52










            • Using the chain rule. A more simple way is to use the coordinates and the fact that for a vector $x=(x_1, dots, x_n)$ (in dimension $n$), you have $Vert x Vert = sqrt{x_1^2 + dots +x_n^2}$
              – mathcounterexamples.net
              Jan 4 at 21:56




















            • Could you explain how you got that?
              – icesk8er
              Jan 4 at 21:52










            • Using the chain rule. A more simple way is to use the coordinates and the fact that for a vector $x=(x_1, dots, x_n)$ (in dimension $n$), you have $Vert x Vert = sqrt{x_1^2 + dots +x_n^2}$
              – mathcounterexamples.net
              Jan 4 at 21:56


















            Could you explain how you got that?
            – icesk8er
            Jan 4 at 21:52




            Could you explain how you got that?
            – icesk8er
            Jan 4 at 21:52












            Using the chain rule. A more simple way is to use the coordinates and the fact that for a vector $x=(x_1, dots, x_n)$ (in dimension $n$), you have $Vert x Vert = sqrt{x_1^2 + dots +x_n^2}$
            – mathcounterexamples.net
            Jan 4 at 21:56






            Using the chain rule. A more simple way is to use the coordinates and the fact that for a vector $x=(x_1, dots, x_n)$ (in dimension $n$), you have $Vert x Vert = sqrt{x_1^2 + dots +x_n^2}$
            – mathcounterexamples.net
            Jan 4 at 21:56












            icesk8er is a new contributor. Be nice, and check out our Code of Conduct.










            draft saved

            draft discarded


















            icesk8er is a new contributor. Be nice, and check out our Code of Conduct.













            icesk8er is a new contributor. Be nice, and check out our Code of Conduct.












            icesk8er is a new contributor. Be nice, and check out our Code of Conduct.
















            Thanks for contributing an answer to Mathematics Stack Exchange!


            • Please be sure to answer the question. Provide details and share your research!

            But avoid



            • Asking for help, clarification, or responding to other answers.

            • Making statements based on opinion; back them up with references or personal experience.


            Use MathJax to format equations. MathJax reference.


            To learn more, see our tips on writing great answers.




            draft saved


            draft discarded














            StackExchange.ready(
            function () {
            StackExchange.openid.initPostLogin('.new-post-login', 'https%3a%2f%2fmath.stackexchange.com%2fquestions%2f3062135%2ftaking-a-derivative-of-a-magnitude-of-a-vector%23new-answer', 'question_page');
            }
            );

            Post as a guest















            Required, but never shown





















































            Required, but never shown














            Required, but never shown












            Required, but never shown







            Required, but never shown

































            Required, but never shown














            Required, but never shown












            Required, but never shown







            Required, but never shown







            Popular posts from this blog

            An IMO inspired problem

            Management

            Has there ever been an instance of an active nuclear power plant within or near a war zone?